LSAT and Law School Admissions Forum

Get expert LSAT preparation and law school admissions advice from PowerScore Test Preparation.

 Administrator
PowerScore Staff
  • PowerScore Staff
  • Posts: 8919
  • Joined: Feb 02, 2011
|
#35655
Complete Question Explanation

Flaw in the Reasoning. The correct answer choice is (A)

This stimulus is a conditional argument. It can be diagrammed as follows:

Conditional argument: Majority favor proposal (MFP) :arrow: Airport

Premise: MFP unlikely

Conclusion: Airport unlikely

There are two issues of note here. First, the argument correctly limits the scope of the conclusion to
the strength of the premise. If a given premise is only unlikely but not impossible, the conclusion
cannot be stronger than unlikely. An inference is only as strong as its weakest link.

Second, the argument confuses sufficient and necessary conditions. MFP is not required for airport;
it only ensures that the airport will be built. But even if a majority of the city’s resident are unlikely
to favor the proposal, another more compelling for building the airport may exist. This is a mistaken
reversal and cannot be used to reach the stated conclusion.

Answer choice (A): This is the correct answer choice. When answering flaw questions that clearly
contain mistaken conditional reasoning, look for terms such as sufficient, necessary, and their
synonyms (needed, ensured, etc.). This answer contains both “sufficient” and “necessary”, which
means it is a clear contender. Now we must ensure that it correctly described the flawed pattern of
reasoning in the stimulus. “A sufficient condition for the airport’s being built” refers to MFP, and the
argument does treat that condition as though it needed to be met or else the airport was unlikely to be
built.

Answer choice (B): The conclusion is not that something must be true, but that something is unlikely
to occur. Also, the argument does not depend on most of Dalton’s residents believing that the airport
will be built, but that most people are unlikely to favor its construction.

Answer choice (C): The argument does reason from the basis that a certain event is unlikely to
occur, but does not conclude that the same event will definitely not occur. For the argument to follow
this pattern of reasoning, the conclusion would have to be something to the effect of, “Since most
residents probably won’t favor the proposal, most residents will not favor the proposal.”

Answer choice (D): Whether or not people living near Dalton would favor the proposal is irrelevant,
since the condition relationship only discusses the majority of Dalton’s residents. The argument does
not need to addresses the reaction of people living near Dalton and is therefore not flawed for failing
to consider it.

Answer choice (E): As always, be careful to pick answers that are both accurate and relevant. It
is accurate that the argument overlooks the possible economy benefit of the airport, but this is not
relevant. Given our premises, the author can draw a perfectly sound conclusion about the likelihood
of the airport being built without referring to any of the pros or cons of the proposal. The author’s
brief reference to perceived noise problems is an attempt to explain the predicted reaction of Dalton’s
residents rather than an accounting of the proposal’s drawbacks.
 SLF
  • Posts: 40
  • Joined: Oct 01, 2013
|
#13092
With regard to LSAT #61, Section #4, Question #11, I need help understanding why this stimulus has the flaw of treating a sufficient condition as a necessary condition.

As I understand things, the stimulus provides this conditional rule:

majority of residents favor proposal (sufficient) :arrow: airport built (necessary)

The contrapositive of this would then be:

!airport built :arrow: !majority of residents favor proposal

Now, from a real-world perspective, I understand that even if all the residents wanted an airport built, that does not mean they have the "clout" to make that happen...and conversely, even if none of them wanted an airport, that does not mean they have the "clout" to stop an airport from being built.

But, setting the real-world aside...and using ONLY this stimulus as LSAC gives it...a rule has been given (defined above)...and the stimulus proceeds to tell that the sufficient part is unlikely to happen...and therefore, the necessary part is also unlikely to happen.

Please help!
 Lucas Moreau
PowerScore Staff
  • PowerScore Staff
  • Posts: 216
  • Joined: Dec 13, 2012
|
#13099
Hello, SLF,

Your diagram is accurate. The trick with this question is it's making a Mistaken Negation. It's going from:

Majority favor proposal :arrow: Airport built

To:

Not!Majority favor proposal :arrow: Not!Airport built

It's mixing up the sufficient and necessary, which is at the heart of all Mistaken Negation or Mistaken Reversal errors.

Just because the majority of the population favoring the proposal is sufficient to see the airport built doesn't mean that's the only way for the airport to be built. Some rich developer could swoop into town and bankroll it on his own, for example. So the lack of a majority does not necessarily doom the airport.

Hope that helps,
Lucas Moreau
PowerScore
 SLF
  • Posts: 40
  • Joined: Oct 01, 2013
|
#13102
Thanks so much Lucas. That is a very helpful response.
 mpoulson
  • Posts: 148
  • Joined: Mar 25, 2016
|
#29043
Hello,

I initially chose C. Is this incorrect because the conclusion says that the airport is unlikely to be built while C claims definitively "that the event will not occur"?

V/r,

Micah
 Claire Horan
PowerScore Staff
  • PowerScore Staff
  • Posts: 408
  • Joined: Apr 18, 2016
|
#29132
Yes, that's a good way of putting it, Micah.
 gmosquera42
  • Posts: 8
  • Joined: Feb 10, 2017
|
#42012
How can you determine when something is not a conditional statement? I was trying to create a condition based on the second premise.


create noise problem--> Unlikely MFP
 Claire Horan
PowerScore Staff
  • PowerScore Staff
  • Posts: 408
  • Joined: Apr 18, 2016
|
#42132
Hi gmosquera42,

That's a good question. Let's say A is a sufficient condition and B is a necessary condition.

A :arrow: B

I should be able to write a sentence that says, "If A, then B."

The statement that you tried to make into an SN cannot be formulated that way because it states an assertion rather than a conditional relationship. Your statement asserted that a majority of residents believed that building the airport would create a noise problem.

The conditional you diagramed—
create noise problem--> Unlikely MFP
—could be read as "If the majority believes that building the airport will create a noise problem, it is unlikely that the majority will favor the proposal." Notice that this sentence doesn't say what the majority believes or doesn't believe. If you don't know what the majority believes, you do not know anything about the likelihood of the majority favoring the proposal.

If you still feel confused, I encourage you to review Powerscore's explanations of Sufficient & Necessary as you may be losing the forest for the trees. Good luck!
 ShannonOh22
  • Posts: 70
  • Joined: Aug 15, 2019
|
#71366
Administrator wrote:Complete Question Explanation

Second, the argument confuses sufficient and necessary conditions. MFP is not required for airport;
it only ensures that the airport will be built. But even if a majority of the city’s resident are unlikely
to favor the proposal, another more compelling for building the airport may exist. This is a mistaken
reversal and cannot be used to reach the stated conclusion.
Isn't the flaw here a Mistaken Negation, not a Mistaken Reversal??
 Paul Marsh
PowerScore Staff
  • PowerScore Staff
  • Posts: 290
  • Joined: Oct 15, 2019
|
#71550
Hi Shannon! You are right, the flaw here is a Mistaken Negation.

However, if you're a bit confused about the relationship between Mistaken Negation and Mistaken Reversal, read on! The correct answer choice (A) for this problem correctly describes that the argument "treats a sufficient condition...as a necessary condition." That is an accurate description of a Mistaken Negation, which is why it's the correct answer. However, it's also an accurate description of a Mistaken Reversal. Why? Because Mistaken Negations and Mistaken Reversals are logically identical. One is just the contrapositive of the other. Watch. Let's say we have a simple conditional:

A :arrow: B

The Mistaken Reversal of that conditional is

B :arrow: A

The Mistaken Negation of that conditional is

A :arrow: B

But notice that

A :arrow: B

is just the contrapositive of

B :arrow: A

So although they may look different at first, Mistaken Negations and Mistaken Reversals are actually the same thing. They are just logical contrapositives of the same flaw: the confusion of sufficient and necessary conditions. Pretty neat, huh? Once you're comfortable with this, then it doesn't really matter if the flaw is characterized as a Mistaken Negation or a Mistaken Reversal. All that matters is whether the argument confused sufficient and necessary conditions. Hope that helps!

Get the most out of your LSAT Prep Plus subscription.

Analyze and track your performance with our Testing and Analytics Package.